12th Class Physics Wave Optics / तरंग प्रकाशिकी Question Bank Assertion And Reason (MCQs) - Wave Optics

  • question_answer
    Assertion (A): Fringes of interference pattern produced by blue light is narrower than that produced by red light.
    Reason (R): In Young's double slit experiment, fringe width = \[\lambda \operatorname{D}/d\]

    A) Both A and R are true and R is the correct explanation of A

    B) Both A and R are true but R is NOT the correct explanation of A

    C) A is true but R is false

    D) A is false and R is true

    Correct Answer: C

    Solution :

    Option [c] is correct.
    Explanation: Fringes of interference pattern produced by blue light is narrower than that produced by red light. The assertion is true.
    Fringe width = \[\lambda \]D/d. Since blue light has wavelength smaller than that of red light, blue light produces narrower fringes. So, reason is also true and explains the assertion.


You need to login to perform this action.
You will be redirected in 3 sec spinner